Writeup: IMO 2004 SL A2
Hey everyone! I haven't posted here in a while because I've been busy with finals and starting my summer research, but I'm glad I finally have time to sit down and revive this blog again. Here's my solution for A2 on the 2004 IMO Shortlist.
Problem Statement:
Let , , , ... be an infinite sequence of integers satisfying the equation for all , where and are two different positive integers.
Can this sequence , , , ... be bounded?
My Solution:
First, a couple facts to note:
- All must be nonnegative, since they are the absolute value of a number.
- We can either have or . From the first fact (above), if , we must have The latter equivalence makes negative.
- It is impossible to have , and it is also impossible to have . What follows is the proof of this fact.
Proof:
First, we prove it's impossible to have . For the sake of contradiction, assume there exists such that . Then there exists minimal with . We can assume since , since , and since .
Since , this would imply that , which would imply that since is positive, which would imply the existence of a smaller such that . Contradiction!
Now, we proceed with the proof. Let . Note that if is finite, then such that , . If , then we must have for all , which obviously diverges and is not bounded.
Now, assume that is instead infinite. Number the in from smallest to largest as . Then note that since if , then .
Now note that is a strictly increasing quantity.
Proof: Note that such that , we must have . Note, also that This implies that . Since the integers are closed under addition, this means that we must have , so must be unbounded and therefore is unbounded.